You are on page 1of 45

CHAPTER 2 Solutions to Assignment Problems

Assignment 2.1:
a.
Freida, Incorporated
Income Statement
For the Year ending December 31, 2006
Gross sales
Less Returns and allowances

462,720
(10,210)

Net sales
Beginning inventory
+ Materials Purchases
- Ending inventory

452,510
63,210
228,580
(68,390)

Cost of goods sold

223,400

Gross profit
Management salaries
Lease payments
Advertising expenditures
R&D expenditures
Repairs and maintenance costs
Depreciation

229,110
17,950
39,270
12,930
4,890
2,910
12,510

Operating profit
Interest expense

138,650
17,090

Earnings before taxes


Taxes

121,560
3,270

Net income

118,290

b. Net profit margin = net income/net sales = 118,290/452,510 = 26.14%


c. Accumulated depreciation = 212,820 + 12,510 = $225,330.

Assignment 2.2:
a.
Windcharter Company
Balance Sheet
For the Year ending December 31, 2006
Cash
Accounts receivable, net
Inventories

17,600
105,770
136,500

ST bank loans
Accounts payable
Accrued expenses

Current assets

259,870

Current portion LT Debt

Gross fixed assets

284,950

Current liabilities
Long-term debt

99,330
134,300

Total liabilities

233,630

Less Accumulated depreciation

(82,310)

Net fixed assets

202,640

Total assets

462,510

Preferred stock
Common stock ($0.20 par)
Additional paid in capital
Retained earnings
Total liabilities & net worth

32,570
50,830
11,850
4,080

8,000
60,000
71,600
89,280
462,510

b. Assume for this problem that the number given for Net income is actually Net income
available to common stockholders (that is, reported Net income minus preferred dividends).
Thus, Annual addition to Retained earnings = Net income available to common stockholders
Common stock dividends paid.
Thus, Common stock dividends paid = Net income Addition to (i.e., change in) Retained
earnings.
Common stock dividends paid = 25,400 (89,280 79,880) = 16,000.
Dividends per share = Dividends divided by number of shares outstanding.
Number of shares outstanding = Common Stock divided by par value per share
= 60,000/.20 = 300,000.
Dividends per share = $0.053 or 5.3 cents per share.
c. Cash spent on new plant and equipment = Depreciation for the year as listed on the income
statement plus the change in net fixed assets.
Thus, Cash spent on fixed assets in 1999 = 10,260 + (202,640 184660) = $28,240.

Assignment 2.3:
1. $4,055,740
$4,100,144
Although Gross sales decreased by 2 percent, net sales (the only number reported) show an
increase of 1.1 percent. Since Returns and allowances are estimates made by management,
there is some chance that the growth in sales conclusion may be misleading.
2. Cost of goods sold = Beginning inventory + Purchases Ending inventory.
a. $174,300
b. $218,500
3. First solve for accumulated depreciation:
2005
2006
Gross fixed assets
3,200,000
4,620,000
Less Accumulated depreciation
(1,280,000)
(1,540,000)
Net fixed assets
1,920,000
3,080,000
Depreciation expense = change in accumulated depreciation = $260,000.

CHAPTER 2 Answers to Additional Problems and Questions


Problems:
1. Look up definitions in chapter
2. EBIT = $15,000; Net income = $6,500
3. a. $400
b. $100
c. $35
4. Gross profit = $3,120,000; Ending Inventory = 850,000.
5.
Gross Sales
-Returns
Net Sales
-COGS
Gross profit
- Operating expenses
- Depreciation
Operating income
- Interest
EBT
- Taxes
Net income

5,000,000
100,000
4,900,000
3,000,000 (250,000 x $12 per unit)
1,900,000
1,000,000
100,000
800,000
500,000
300,000
120,000
180,000

6.
Sales
Cost of goods sold

32,000
19,200

Gross profit
Operating expenses
Depreciation

12,800
4,000
3,000

Operating profit
Interest expense

5,800
2,800

EBT
Taxes

3,000
900

Net income

7.
Inventory current asset
Accounts receivable current asset
Long-term debt noncurrent liability
Common stock (par value) - equity
Plant and equipment noncurrent asset
Cash current asset
8. Total shareholders equity = $1,450,000.

2,100

Retained earnings - equity


Accounts payable current liability
Accrued wages and taxes current liability
Notes payable (bank loans) current liability
Marketable securities current asset
Prepaid expenses current asset

9. In the following solution, the reported number for Gross fixed assets is assumed to exclude
Leasehold improvements.
Quest-Mar, Incorporated
Balance Sheet
For the Year Ending Decemebr 31, 2006
Cash
Net Accounts receivable
Inventories
Current assets

120
100
190
410

ST Bank loan
Accounts payable
Accrued expenses
Current portion LT Debt

Gross fixed assets (exc. Lease. Imp.)


Leasehold improvements

900
300

Current liabilities
Long-term bank loan

Less Accumulated depreciation


Net fixed assets

Total assets

10. a. $782,200
11. CP LT Debt
LT Debt

(200)
1,000

1,410

b. $54,532,500
2004
280,000
2,240,000

Total liabilities
Common stock ($1.00 par)
Retained earnings

20
90
40
60
210
600
810
400
200

Total stockholder's equity

600

Total liabilities & net worth

1,410

c. $44,465,000
2005
280,000
1,960,000

2006
280,000
1,680,000

12. a. 75,000 shares (note that all figures for this problem are in thousands).
b. (10,450,000 8,700,000) / 75,000 = $23.33
c. $1.12 (total dividends paid by the
company = NI(2006) change in retained earnings = 700,000. Divide this by 625,000
shares outstanding to get $1.12 per share).
13. Total dividends paid = $6,000,000 ; Dividends per share = $1.20
14. a. EPS=$6.14; DPS = $1.89
b. $180,400 c. EPS=$6.01; DPS=$1.76 (assumes
3,000 shares issued typo on problem).
15. a. 5.25
b. Price=8.25
c. 1.57 times
Equity represents shareholder wealth. Book value is historical. Market value is a function of
the expected future cash flows of the company. Book value is approximately what the firms
assets are worth if liquidated. Market value is what the company is worth as a functioning
entity. Thus, this company is worth approximately 1.57 times more as an operating company
than if the company stopped operating and sold off all of its assets. The larger this ratio, the
more successful the company is at creating value in operations.

16. a.
b.
c.
d.
e.
17. a.
b.
c.
d.
e.
f.

Owners equity in 2005 = $140,000; owners equity in 2006 = $144,000


$10,000
$40,000
$4,000
$50,000
Owners equity in 2005 = $100,000; owners equity in 2006 = $110,000
$20,000; $24,000
$10,000
$3,000
$128,000 note this merely equals the change in net fixed assets + depreciation from I/S
+$22,000

CHAPTER 3 Solutions to Assignment Problems


Assignment 3.1:
Net income
+ Depreciation
+ Decrease in Accounts receivable
- Increase in Inventories
- Decrease in Accounts payable
- Decrease in Accruals
Net Cash Flow from Operating Activities

48,000
30,000
21,000
(10,900)
(12,000)
(14,000)
62,100

Purchase of Gross fixed assets*


Net Cash Flow from Investing Activities

(40,000)
(40,000)

Increase in Notes payable


Change in LT Debt
Change in Common Stock
Increase in Capital surplus
- Payment of dividends**
Net Cash Flow from Financing Activities

15,000
6,000
5,000
1,000
(33,200)
(6,200)

Change in Cash

15,900

* Change in Net fixed assets + Depreciation


** Net income - (Change in Retained earnings)

Assignment 3.2:
Net income
+ Depreciation
- Increase in Accounts receivable
- Increase in Inventories
+ Increase in Accounts payable
+ Increase in Accruals
Net Cash Flow from Operating Activities
Purchase of Gross fixed assets*
Net Cash Flow from Investing Activities
Change in LT Debt
Change in Common Stock
- Payment of dividends**
Net Cash Flow from Financing Activities
Change in Cash
* Change in Net fixed assets + Depreciation
** Net income - (Change in Retained earnings)

1,800
1,600
(500)
(300)
1,100
500
4,200
(2,400)
(2,400)
(1,200)
(1,200)
600

Assignment 3.3:
Net income
+ Depreciation*
+ Decrease in Accounts receivable
- Increase in Inventories
+ Increase in Accounts payable
- Decrease in Accruals
Net Cash Flow from Operating Activities
Purchase of Gross fixed assets**
Net Cash Flow from Investing Activities
Change in Notes payable
Change in LT Debt
Change in Common Stock
Change in Capital surplus
- Payment of dividends***
Net Cash Flow from Financing Activities
Change in Cash
* Change in Accumulated depreciation
** Change in Gross fixed assets
*** Net income - (Change in Retained earnings)

32,000
30,000
7,000
(30,900)
8,000
(1,000)
45,100
(40,000)
(40,000)
(3,000)
15,000
2,000
4,000
(24,900)
(6,900)
(1,800)

CHAPTER 3 Answers to Additional Problems and Questions


Problems:
1. Raise prices, reduce growth rate, speed up collections, hold smaller inventory levels, etc.
2. A reduction or non-payment of a dividend, the substitution of a stock dividend for a cash
dividend, late payments to suppliers or other creditors, bounced payroll checks, etc.
3.
Cash
Accounts receivable
Inventories
Total assets

July Balance Sheet


15,000
Debt
35,000
Stock
28,000
Retained earnings
78,000

Total claims

40,000
38,000
78,000

4. The growth in sales slowed to zero percent per year. Note that outflows for this company are
based on the current months sales level. That is, outflows in February are based on February
purchases (which are based on February sales). Outflows in March are based on March
purchases (which are based on March sales). And so on. Conversely, inflows are based on
sales that occurred one month ago. Thus, inflows in February are based on January sales and
inflows in March are based on February sales. In periods of rapid growth, this one-month lag
can cause outflows to exceed inflows even though revenue exceeds costs for that period.
When sales growth slows to zero, the lag effectively disappears since sales in each month are
the same. In this case, inflows minus outflows are essentially the same as revenue minus cost.
5. Depreciation expense (2004) = $2,000.
Depreciation expense (2005) = $4,000.
Depreciation expense (2006) = $2,000.
6. Change in gross fixed assets (2004) = $20,000.
Change in gross fixed assets (2005) = $40,000.
Change in gross fixed assets (2006) = $20,000.
7. Change in Net fixed assets + Depreciation (2004) = 18,000 + 2,000 = $20,000.
Change in Net fixed assets + Depreciation (2005) = 36,000 + 4,000 = $40,000.
Change in Net fixed assets + Depreciation (2006) = 18,000 + 2,000 = $20,000.

8.
Net income
+ Depreciation
- Increase in Accounts receivable
- Increase in Inventories
- Decrease in Accounts payable
Net Cash Flow from Operating Activities
Purchase of Gross fixed assets
Net Cash Flow from Investing Activities
Change in Notes payable
Change in LT Debt
Change in Common Stock
- Payment of dividends
Net Cash Flow from Financing Activities
Change in Cash

26,000
2,000
(4,000)
(2,000)
(2,000)
20,000
(15,000)
(15,000)
3,000
8,000
15,000
(18,000)
8,000
13,000

9. a. Total dividend paid = NI change in R. E. = 3200 (4000-2800) =$2000


b. Change in GFA/purchase = change in NFA + dep = (14,900-13,300)+1800 = $3400
GFA in 2006 = GFA of 2005 + purchase =20,000+ 3400 = $23, 400
10.
Net income
+ Depreciation
+ Decrease in Accounts receivable
- Increase in Inventories
+ Increase in Accounts payable
- Decrease in Accruals
Net Cash Flow from Operating Activities

3,200
1,800
500
(600)
1,800
(400)
6,300

Purchase of Gross fixed assets

(3,400)

Net Cash Flow from Investing Activities

(3,400)

Change in Notes payable


Change in LT Debt
Change in Common Stock
- Payment of dividends

400
(1,300)
600
(2,000)

Net Cash Flow from Financing Activities

(2,300)

Change in Cash

600

6,300 3,400 2,300 = 600 (check)


11. Net cash flow from Operating Activities = $4,500
Net cash flow from Investing Activities = ($12,200)
Net cash flow from Financing Activities = $8,200
12. Net cash flow from Operating Activities = $470,000
Net cash flow from Investing Activities = ($350,000)*
Net cash flow from Financing Activities = ($110,000)
*Note that Long-term investments are an investing activity. An increase in Long-term
investments represents an outflow of cash.
13. a. $94,200

b. $113,500

c. $19,300

14. NCF Operating Activities = $22,900; NCF Investing Activities = ($23,200); NCF Financing
Activities = $100.
15. a. $1.96 million (compute net income and then add back depreciation, since depreciation is
not a cash flow).
b. $400,000 (note that when depreciation increases, taxes decrease. This is why depreciation,
although not a cash flow, can affect net cash flow from operations. Specifically, by
changing the amount of taxes a company owes.
16. a. ($6,000) - note that change in cash and marketable securities = ($4,000)
b. ($2,000)
c. ($28,000)
d. $26,000

The sum of these three cash flows = -$4000, which is the change in cash and marketable
securities. Marketable securities, as explained in chapter 2, are an alternative form of cash.
They are very short-term investments that generate a positive return; they can be converted
into cash very quickly. Essentially, cash that earns a return.

CHAPTER 4 Solutions to Assignment Problems


Assignment 4.1:
1. a.
2005
Net sales
Cost of goods sold
Gross profit
Operating expenses
Operating income
Income taxes
Net income

2006

100.0%
62.9%
37.1%
27.1%
10.0%
4.2%
5.8%

100.0%
64.1%
35.9%
26.5%
9.5%
4.1%
5.4%

Note that the gross profit margin deteriorates by 1.2% and Operating profit margin deteriorates
by 0.5%. Company has been able to offset some of the gross margin deterioration by controlling
operating expenses either real or manipulation of discretionary expenses.
2. (Note that some of the ratios you compute for this problem will look nonsensical. This is
done on purpose to stress computation of the ratios. Hopefully you will note that a company
with some of these ratios in particular ACP and Inventory turnover, would probably be in
very poor shape).
Current ratio
Average collection period
Inventory turnover ratio
Total asset turnover ratio
Payables ratio
Debt ratio
Cash conversion cycle
Net profit margin
Return on equity

2005
2.3
288.2
.40
.28
488.3
57%
715.5
5.8%
3.8%

2006
2.4
365
.39
.24
491.8
35%
805.1
5.4%
3.2%

Assignment 4.2:
a. Total dividends = Net income Change in Retained earnings = 1,600,000 600,000 =
1,000,000.
b. Number of shares = Common stock/ Par value per share = 1,500,000/.25 = 6,000,000 shares.
c. Number of shares = Common stock/ Par value per share = 1,900,000/.25 = 7,600,000 shares.
d. Dividends per share = Total dividends / Number of shares = 1,000,000/7,600,000 = 13 cents.
e. ROE = Net income / Total Equity = 1,400,000/ (1,500,000 + 1,000,000 + 1,000,000) = 40%.
f. ROE = Net income / Total Equity = 1,600,000/ (1,600,000 + 2,400,000 + 1,900,000) = 27%.
g. ROE (Dupont) = Net income/Sales x Sales/Total assets x Total assets/Total equity.
ROE (2002-Dupont) = .200 x 1.167 x 1.714 = .40 = 40%
ROE (2003-Dupont) = .178 x 0.900 x 1.695 = .27 = 27%
Net profit margin declined in 2003 by 11% (from 20% to 17.8%), Total asset turnover ratio
declined by 23% (from 1.714 to 0.900) and the equity multiplier declined by 1.1%. The
major reason for the deterioration in ROE is a decrease in the efficiency of asset utilization.
Secondary reason is a decline in profitability.
Assignment 4.3:
The best approach to do these fill in the balance sheet type problems is to use any given values
(in this case, Total assets = $1,000,000) to compute as many related values as possible.
For example, given that Assets (A) = 1,000,000, then from the fact that the Debt ratio (Total
liabilities/Total assets) = 60%, Total liabilities = 600,000. Since Total assets = Total liabilities +
Total equity (basic accounting relationship), Total equity must be 400,000.
On the given balance sheet, Total liabilities = Accounts payable + Long-term debt, and since
Long-term debt is given as 180,000, Accounts payable = 420,000. Also, on the given balance
sheet, since Total equity = Common stock + Retained earnings, and since Retained earnings is
given as 200,000, Common stock = 200,000.
Next use, Average collection period = 45 days = (Accounts receivable/Sales) x 360. From Total
asset turnover = Sales/Total assets = 2.0, Sales = 2,000,000. Thus,
45 = (Accounts receivable/2,000,000) x 360, so Accounts receivable = 250,000.
Then use, Inventory turnover ratio = 5 = COGS/Inventory. From Gross profit margin = 25% and
Sales = 2,000,000, COGS = 1,500,000. Thus,
5 = 1,500,000/Inventory, so Inventory = 300,000.
Then use Quick ratio = 1.0 = (Current assets - Inventory)/(Current liabilities). Note that Current
assets for this balance sheet = Cash + Accounts receivable + Inventory, so Current assets
Inventory = Cash + Accounts receivable. Also note that for this balance sheet, Current liabilities
= Accounts payable. Thus,

1 = (Cash + 250,000)/(420,000), so Cash = 170,000.


Finally, Cash + Accounts receivable + Inventory + Fixed assets = Total assets. Thus, Fixed
assets = 280,000.
Summarizing:
Cash
Accounts receivable
Inventories
Current assets
Fixed assets
Total assets

170,000
250,000
300,000
720,000
280,000
1,000,000

Accounts payable
Current liabilities
Long-term debt
Total liabilities
Common stock
Retained earnings
Total claims

420,000
420,000
180,000
600,000
200,000
200,000
1,000,000

2. Sales = 2,000,000; COGS = 1,500,000


3. CCC

= Inventory conversion period + Average collection period Payables period


i. = 72 + 45 100.8 = 16.2 days.

4. ROE

= Net profit margin x Total asset turnover ratio x (Total assets/Equity)


= .10 x 2.0 x (1,000,000/400,000) = .50 = 50%

CHAPTER 4 Answers to Additional Problems and Questions


Problems:
1. A grocery store. Would you want to shop at a grocery store that had an inventory turnover
ratio of 2? That is, all items (fruit, meat, vegetables, etc.) turned over (i.e., sold) on average
only once every 6 months!
2. High inflation can distort the relationship between book values of assets and true economic
values. For example, if a firm has land on its balance sheet listed in value at $1,000 and
inflation in land values in running at 40 percent annually, 5 years later the market value of
the land would be worth approximately $5,400 but would only be listed on the balance sheet
as being worth $1,000. A similar distortion would apply to plant and equipment.
For short-term assets, however, values would, in general, adjust with inflation. If inventory
turns over relatively rapidly, each turnover would adjust inventory values upward. The same
would be true of accounts receivable. Thus, the relationship between the book value of shortterm assets and long term assets will be distorted. Finally, since the income statement is a
flow measure, high inflation usually affects all revenues and costs in a similar manner.
Assume that a firm has revenue of $1,000 and costs (total) of $800 and thus profit of $200. If
inflation is 50 percent and it impacts revenue and costs in a similar manner, then revenue will
grow to $1,500 (due solely to inflation) and costs will increase to $1,200 causing profit to
grow to $300. Thus inflation can cause a dramatic increase in net income (even more

pronounced if some costs are fixed). Now consider the impact of this increase on a ratio such
as ROA (recall from above that assets do not necessarily grow in line with inflation).
3. The current ratio is CA/CL. A declining ratio can either be due to a relative decline in CA or
a relative increase in CL. Since the inventory turnover ratio and the ACP are constant
through time, inventory and accounts receivable appear to be relatively stable. The increase
in the Payables period would indicate a relatively rapidly increasing Accounts payable value,
which would cause a relative increase in CL and therefore a decline in the current ratio. The
analyst should investigate the causes of the increase in AP perhaps a new supplier to the firm
offers more lax credit terms than the previous supplier.
4. 72%; 14.4%; 45%
5. CA = 2(CL) = 200,000. (CA - Inv)/100,000 = 1.5; thus, Inv = 50,000.
6. HINT: For problems where you are only given ratios and NO actual dollar amounts, assume
that assets are $100 and solve for all other numbers - then find the answer.
Thus, assume A = 100.
From ROA = NI/A; .20 = NI/100; thus, NI = 20.
Then, from ROE = NI/E; .50 = 20/E; thus, E = 40.
Then, from the definition, A = D + E; 100 = D + 40; thus, D = 60. (NOTE: this relationship
can be used for most of these types of problems).
Finally, compute the debt ratio = D/A = 60/100 = .60 = 60%.
7. Assume A = 100. Then from S/A = 4, S = 400. Then from D/A = .20, D = 20. Then from A =
D + E, E = 80. Then from NI/E = .10, NI = 8.
Finally, compute net profit margin = NI/S = 8/400 = 2%.
8. For this problem, you are given a dollar amount, so DO NOT assume that A = 100. Instead,
from ACP =(AR/S) x 360; 20 = (1,000/S) x 360; thus, S = 18,000. Then from NI/S = .05, NI
= 900. Then from S/A = 2, A = 9,000. Then from D/A = .75, D = 6,750. Then from A = D +
E, E = 2,250. Finally, ROE = NI/E = 900/2,250 = 40%.
9. Net profit margin = 8%; Debt ratio = 52%
10. ROA = 12.5%; ROE = 16.7%
11. Net profit margin = 8%; Debt ratio = 52%
We know that if we solve for x in (1950+x)/(800+x) = 2, we get the maximum amount by
which inventory = bank debt =x can increase without violating a current ratio of 2.Solving
for x we get, x = 350. In this case, Inv will be 1,050. Thus, (CA-Inv)/CL = (1950+350-1050)/
(800+350), and Quick ratio = 1.087.

12. Assume A = 100. Then D = 75 and E = 25 (recall A = D + E). Thus, Equity multiplier = A/E
= 100/25 = 4.
13. Answers assume a 360-day year.
a. $100
b. $450
c. $150
d. 5.7%
e. $400
f. $500
14. ROE = 42% (since all equity financed, equity = total assets)
15. Max Increase in debt = 1.4
16. B
17. From ACP = (ARx360)/S, 60 = (150,000x360)/S, S = 900,000. From NI/S = .04, NI =
36,000. From D/A = .64 and A = 3,000,000, D = 1,920,000. From A = D + E, E = 1,080,000.
Then, ROE = NI/E = .033 = 3.3%.
18. Note that CA = Cash & Mkt. Sec. + AR + Inv. and Inventory turnover here is defined as
S/Inv. From CA/CL = 3 and CA = 810,000, CL = 270,000. From (CA-Inv)/CL = 1.4 and
note above and Cash & Mkt. Sec. = 120,000, AR = 258,000. From note above, now Inv =
810,000 - 120,000 - 258,000 = 432,000. From S/Inv = 6, S = 2,592,000. From ACP =
(ARx360)/S, ACP = 35.8 days.
19. From NI/S = .06, NI = 120,000. From tax rate = .40 and from NI = EBT - (.40)x(EBT), EBT
= 200,000. To check, note that 200,000 x .4 = taxes of 80,000 and EBT - taxes = 120,000.
From EBT = EBIT - Interest, EBIT = 220,000. Thus, TIE = EBIT/Interest = 11 times.
20. Assume that currently A = 100. Then, currently, from debt ratio = .5, D = 50 and E = 50.
From S/A = .25, S = 25. From NI/S = .10, NI = 2.5. Using these numbers note that ROE =
5% (they want to double the current ROE from 5% to 10%) checks out. For new, assume that
S and A remain the same such that S/A will remain the same. Thus, S = 25 and A = 100.
From NI/S = .14, NI = 3.5 For ROE = NI/E to be .10, E must be 35. From A = D + E, D = 65
and thus, debt ratio = D/A = 65%.
21. Assume A = 100. From D/A = .35, D = 35 and E = 65. From NI/E = .15, NI = 9.75. From
S/A = 2.8, S = 280 Thus, NI/S = .0348 = 3.48%.
22. For these conversion type problems, note that if D = D/E and E = 1, D/E will be the stated
amount. Thus, for the first one, if D = 2.5 and E = 1, then D/E = 2.5. From A = D + E, A =
2.5 + 1 = 3.5. Then, D/A = 2.5/3.5 = .714 = 71.5% and A/E (the equity multiplier) = 3.5/1 =
3.5 times. For the second one, let D = 1 and E = 1 (thus, D/E = 1). Then A = D + E and A =
2. Thus, D/A = .5 and A/E = 2. For the third one, let D = .6 and E = 1. Then, A = 1.6. Thus,
D/A = .375 = 37.5% and A/E = 1.6 times.

23. From NI/S = .075, NI = 150,000.


From NI/E = .24, E = 625,000
From NI/A = .15, A = 1,000,000
Thus, Total liabilities = NP + AP + LT Debt = 375,000.
From S = 2,000,000 and Gross profit = 400,000, COGS = 1,600,000.
From COGS/Inv = 4, Inv = 400,000.
From (APx360)/COGS = 20, AP = 88,889.
From ACP = (ARx360)/S = 40, AR = 222,222.
From CA/CL = 3.8 and CA = AR + Inv = 622,222, CL = 163,743.
From CL = Notes payable + AP, Notes payable = 74,854.
Thus, long-term debt = 375,000 - 74,854 - 88,889 = 211,257.
Finally, A = 1,000,000 = AR + Inv + Fixed Assets.
Thus, Fixed assets = 377,778.
Summarizing, AR = 222,222; Inv = 400,000; Fixed assets = 377,778;
Total assets = 1,000,000; Notes payable = 74,854; AP = 88,889;
Long-term debt = 211, 257; Equity = 625,000;
Total liab & eq. = 1,000,000.
24. See if you can derive the following answer on your own. Hint, compute Accounts receivable
first, and then use the Quick ratio and Current ratio to compute Current liabilities and
Inventories.
Accounts receivable
Inventories
Fixed assets
Total assets

400
800
2800
4000

Current liabilities
Long-term debt
Equity
Total claims

400
2000
1600
4000

CHAPTER 5 Solutions to Assignment Problems


Assignment 5.1:
Balance Sheet
FreshFish, Inc.
Years ending December 31, 2006 and 2007
2006
Cash
$ 20,000
Accounts receivable
10,000
Inventory
80,000
Total Current Assets
$110,000
Net plant and equipment
$430,000
Total assets
$540,000

2007
$ 30,000
15,000
120,000
165,000
645,000
$810,000

Notes payable
Accounts payable to suppliers
Accruals
Total current liabilities
Long-term debt
Common stock ($2.00 par value)
Capital surplus
Retained earnings
Total Liabilities and Equity

15,000
75,000
7,500
97,500
190,000
20,000
150,000
138,350
$595,850

15,000
50,000
5,000
$ 70,000
190,000
20,000
150,000
110,000
$540,000

2. OFN = $810,000 $595,850 = $214,150


3. 540,000(.50) 55,000(.50) 900,000(1.50)(.035)(.60) = 270,000 27,500 28,350
= $214,150
Assignment 5.2:
You can answer this question for all parts using the OFN equation. Note that in that equation, the
last part is the change in retained earnings due to projected profit that is reinvested back into the
firm. This part of the equation is given as:
[(S0 + (g)(S0)](NPM) Div0]
The first part of this equation is projected net income. With some minor mathematical
rearrangement, it can be written as (S0)(1+g)(NPM). The last term in the equation is the amount
of net income that is paid out in dividends. In the equation, it is given as a constant. For
Assignment 6.2, it is stated as a percent of net income (i.e., Tabler pays out 40 percent of all its
annual profit in dividends). If 40% of profit is paid out, then the remainder (i.e., 60%) must be
retained. Mathematically, this can be written as:
(S0)(1+g)(NPM)(.60)
Thus, the OFN equation becomes:
OFN = (TA0)(g) [(L0)(g) + (S0)(1+g)(NPM)(.60)]
And filling in all other given values:
OFN = (1,220,000)(g) [(500,000)(g) + (4,000,000)(1+g)(.05)(.60)]
= 1,220,000g 500,000g 120,000g 120,000
= 600,000g 120,000.

a.
b.
c.
d.
e.
f.

For g = 0% = 0, OFN = -120,000


For g = 10% = .10, OFN = -60,000
For g = 20% = .10, OFN = 0
For g = 30% = .10, OFN = 60,000
For g = 40% = .10, OFN = 120,000
For g = 50% = .10, OFN = 180,000

2. OFN = 0 when g = 20% (see c above).


Assignment 5.3:
a. Earnings per share = Net income divided by number of shares of common stock
outstanding. Net income = (.05)(90,000) = 4,500. Number of shares = Common stock
value of balance sheet divided by par value per share = 60,000/2 = 30,000. Thus,
Earnings per share = 4,500/30,000 = .15 = 15%
b. Dividends = Net income Change in retained earnings = 4,500 (30,000 28,200) =
2,700. Dividend payout ratio = Dividends divided by Net income (that is, the ratio
measures the percent of net income that is paid out in dividends).
Dividend payout ratio = 2,700/4,500 = .60 = 60%.
c. Use the equation: OFN = (TA0)(g) [(L0)(g) + [(S0 + (g)(S0)](NPM) Div0].
OFN = (200,000)(.40) [(60,000)(.40) + [(90,000 + (.40)(90,000)](.05) 4,000]
= 80,000 24,000 2,300
= 53,700.
2. Construct the proforma income statement:
Sales
$330,000
Operating Costs (70%)
231,000
Fixed costs
60,000
EBIT
39,000
Interest
10,000
EBT
29,000
Taxes (40%)
11,600
Net income
17,400
Dividends
9,000
Addition to Retained earnings
8,400
Now, construct the proforma balance sheet:
Current assets
Net fixed assets
Total assets

110,000
55,000
165,000

OFN = 165,000 161,400 = 3,600.

Notes payable
Accounts payable
Accruals
Long-term debt
Equity
Total claims

20,000
22,000
11,000
50,000
58,400
161,400

CHAPTER 5 Answers to Additional Problems and Questions


Problems:
1. Reduce the dividend payout rate or increase the net profit margin.
2. a. and b.
Apr
75,000
350,000
(275,000)

May
100,000
350,000
(250,000)

Begin. Cash + NCF

75,000

(175,000)

Ending ST Loan Balance


Ending Cash Balance

75,000

225,000
50,000

Cash Collections
Cash Expenses
Net Cash Flow (NCF)

Jun
300,000
350,000
(50,000)
275,000
50,000

Jul
500,000
350,000
150,000

Aug
900,000
350,000
550,000

Sep
500,000
350,000
150,000

150,000

600,000

625,000

125,000
50,000

475,000

625,000

NOTE: If ending cash balance is greater than 50,000, the ST loan is zero. If ending cash balance is less than
50,000, the ST loan will grow so as to make ending cash equal to 50,000.

c. Accounts receivable balance (Apr) = 100,000


Accounts receivable balance (May) = 300,000
Accounts receivable balance (Jun) = 500,000
Accounts receivable balance (Jul) = 900,000
Accounts receivable balance (Aug) = 500,000
Accounts receivable balance (Sep) = 200,000
3. Annual Inventory Related Interest Cost (current) = 25 x 500 x .10 = $1,250

Annual Inventory Related Interest Cost (proposed) = 15 x 500 x .10 = $750


So, reduction in annual inventory related interest cost = $750.
Reduction in Rent = $5,000 annual savings.
So, total benefit of proposed change = $5,750
Number of Inventory orders per year (current) = 12
Number of Inventory orders per year (proposed) = 36
Increase in Inventory order cost = (36)($250) = $6,000
Thus, benefit cost = $5,750 - $6,000 = ($250) DO NOT CHANGE POLICY.

4.
COFFYS
Proforma Balance Sheet
For the Year Ending June 30, 2004
Cash
Accounts receivable
Inventories
Current Assets
Gross fixed assets
Accumulated dep.
Net fixed assets

$14,000
11,200
100,800
126,000

Notes payable
Accounts payable
Accruals
Current Liabilities

$10,000
35,000
7,000
52,000

1,148,000
(84,000)
1,064,000

Long term bank loan


Common stock
Retained earnings

400,000
200,000
226,000

Total assets

1,190,000
878,000

Total Liab. & equity

All asset accounts grow by 40% and accounts payable and accruals (i.e., spontaneous liabilities)
grow by 40%. Financing variable (i.e., Notes payable, Long-term bank loan and Common stock)
values stay fixed. The change in Retained earnings is due to net income that is not paid in
dividends. Specifically,
Change in RE = (2003 Sales)(1 + Projected growth rate)(Net profit margin) Dividends
= (800,000)(1.4)(.05) 40,000
= 16,000
And thus proforma Retained earnings = 210,000 + 16,000 = 226,000.
Outside funds needed = 1,190,000 878,000 = 312,000.
Check:
OFN = (TA0)(g) [(L0)(g) + [(S0 + (g)(S0)](NPM) Div0]
= (850,000)(.40) [(30,000)(.40) + [(800,000 + (.40)(800,000)](.05) 40,000
= 340,000 [12,000 + 16,000]
= 312,000.
5.
COFFYS
Proforma Balance Sheet
For the Year Ending June 30, 2004
Cash
Accounts receivable
Inventories
Current Assets
Gross fixed assets
Accumulated dep.
Net fixed assets

Total assets

$14,000
11,200
100,800
126,000

Notes payable
Accounts payable
Accruals
Current Liabilities

$ 8,400
35,000
7,000
50,400

1,148,000
(84,000)
1,064,000

Long term bank loan


Common stock
Retained earnings

400,000
200,000
226,000

1,190,000

Total Liab. & equity

876,400

All asset accounts grow by 40% and accounts payable and accruals (i.e., spontaneous liabilities)
grow by 40%.
To solve for Notes payable, use the fact that the current ratio will be 2.5. Since you have
projected current assets = 126,000, this implies that projected current liabilities will be 50,400.
With Accounts payable = 35,000 and Accruals = 7,000 (due to spontaneous liabilities grow by
40%), Notes payable = 52,000 35,000 7,000 = 8,400.
The remaining financing variable (i.e., Long-term bank loan and Common stock) values stay
fixed. The change in Retained earnings is due to net income that is not paid in dividends.
Specifically,
Change in RE = (2003 Sales)(1 + Projected growth rate)(Net profit margin) Dividends
= (800,000)(1.4)(.05) 40,000
= 16,000
And thus proforma Retained earnings = 210,000 + 16,000 = 226,000.
Long term outside funds needed = 1,190,000 876,400 = 313,600. (This money will come from
some combination of additional long-term debt and additional common stock).
6.
COFFYS
Proforma Balance Sheet
For the Year Ending June 30, 2004
Cash
Accounts receivable
Inventories
Current Assets
Gross fixed assets
Accumulated dep.
Net fixed assets

Total assets

$14,000
11,200
100,800
126,000

Notes payable
Accounts payable
Accruals
Current Liabilities

$21,000
35,000
7,000
63,000

1,148,000
(84,000)
1,064,000

Long term bank loan


Common stock
Retained earnings

413,000
200,000
226,000

1,190,000
902,000

Total Liab. & equity

All asset accounts grow by 40% and accounts payable and accruals (i.e., spontaneous liabilities)
grow by 40%.
To solve for Notes payable, use the fact that the current ratio will be 2.0. Since you have
projected current assets = 126,000, this implies that projected current liabilities will be 63,000.
With Accounts payable = 35,000 and Accruals = 7,000 (due to spontaneous liabilities grow by
40%), Notes payable = 63,000 35,000 7,000 = 21,000.
To solve for Long-term bank loan, use the fact that the debt ratio will be 40 percent. The debt
ratio = total liabilities divided by total assets. With total assets projected to be 1,190,000, total
liabilities will be (.40)(1,190,000) = 476,000. Since Current liabilities = 63,000, Long-term bank
loan must be 476,000 63,000 = 413,000.

The remaining financing variable (i.e., Common stock) value stay fixed. The change in Retained
earnings is due to net income that is not paid in dividends. Specifically,
Change in RE = (2003 Sales)(1 + Projected growth rate)(Net profit margin) Dividends
= (800,000)(1.4)(.05) 40,000
= 16,000
And thus proforma Retained earnings = 210,000 + 16,000 = 226,000.
Money needed from selling additional common stock = 1,190,000 902,000 = 288,000.
7. a. $101,000

b. - $24,000
(TA = 1250000*1.15 =1437,500, 60% of which is debt, Total equity needed = 0.4 of
1437,500 = 575,000. Now addition to R. E = 49,000. Total R. E = 300,000+49,000 =
349,000. Total equity fund available = (50,000+200,000+349,000) = 599,000. Therefore,
common stock needed (575,000-599,000) = -24,000
8. $130,400
9. $112,500
10. a. $150,000
b. $137,400
c. $124,800
11.

(a)
Cash
Acc. receivable
Inventory
Net Prop. & Plant
Total Assets

2003 Actual
400,000
900,000
1,200,000
2,500,000
5,000,000

Acc. payable
Long term debt
Common Stock
Retain earnings
Tot. Liab. & Equity

800,000
1,500,000
1,800,000
900,000
5,000,000

2004 Proforma
500,000
1,125,000
1,500,000
3,125,000
6,250,000
1,000,000
1,500,000
1,800,000
1,140,000
5,440,000

Total Outside Funds Needed = 810,000


(b)
Cash
Acc. receivable
Inventory
Net Prop. & Plant
Total Assets

2003 Actual
400,000
900,000
1,200,000
2,500,000
5,000,000

Acc. payable
Long term debt
Common Stock
Retain. earnings

800,000
1,500,000
1,800,000
900,000

2004 Proforma
500,000
1,125,000
1,500,000
3,125,000
6,250,000
1,000,000
2,750,000
1,800,000
1,140,000

Tot. Liab. and Equity


5,000,000
Additional Common Stock Needed =(440,000)
12. a. Total Outside Funds Needed = 2,235,000

b. Total Outside Funds Needed = 885,000

6,690,000

CHAPTER 6 Solutions to Assignment Problems


Assignment 6.1:
1. FV = PV (1+r). Let FV = 2 and PV =1. Thus, 2 = 1(1+r). When you solve for r, you get r=1.
Expressed as a percent, the interest rate is 100%.
2. FV = PV (1+r); 7397 = PV(1+.0439) or PV = 7297/1.0439; Solve PV = 7,085.93.
3. FV = PV (1+r); 13000 = PV(1.08) or PV = 13000/1.08; Solve for PV = 12,037.04.
4. Note that you will only need 25000 4500 = 20,500 one year from today. Thus, FV = PV
(1+r); 20500 = PV(1.075) or PV = 20500/1.075; Solve for PV = 19,069.77
5. FV = PV (1+r); 14739 = PV(1.12) or PV = 14739/1.12; Solve for PV = 13,159.82.
Assignment 6.2:
1. FVn = PV (1+r)n. Let FV = 2 and PV =1. Thus, 2 = 1(1+r)2 or 2 = (1+r). When you solve for
r, you get r=.4142. Expressed as a percent, the interest rate is 41.42%.
2. PV = FV1(1/1+r)1 + FV2(1/1+r)2 ; PV = 3200(1/1.06)1 + 7300(1/1.06)2 ; PV = 3018.87 +
6496.97 = 9,515.84.
3. Note that the first deposit will grow for one year that is, it will grow to become 7448 (1.07)
= 7969.36. When you add the extra 2476, you will have a total of 10,445.36 in your account.
4. Continuing from #3, the 10,445.36 will grow again by 7 percent to be 10445.367(1.07) =
11,176.54.
5. 15000 = X(1.12)2 + X(1.12)1 = X [(1.12)2 + (1.12)1] = X[2.3744]. Thus, X = 6,317.39.
Assignment 6.3:
1. FV2 = Deposit0 (1+r)2 + Deposit1 (1+r)1. (Note that the deposit made today (at t=0) will earn
interest for 2 years and the deposit made one year from today will earn interest for 1 year).
Thus, 4000 = 8000(1.04)2 + Deposit (1.04)1; 4000 = 8652 + Deposit (1.04). Solve for Deposit
and you get Deposit = -4,473.85. The negative sign implies that you will withdraw this
amount at the end of year one.
2. The question is what is the present value of the investment. That is, what is the present value
of 6500 one year from today + 5000 two years from today. We then compare the value of the
investment with the cost. If value > cost, you should buy. If value < cost, you should not buy.
PV = FV1(1/1+r)1 + FV2(1/1+r)2 ; PV = 6500(1/1.12)1 + 5000(1/1.12)2 ; PV = 5803.57 +
3958.97 = 9,789.54. Do not make investment.
3. 538 = 500 (1+r)1; 1.0760 = 1+r; r = .0760 = 7.6%
4. (1 + rnominal) = (1 + rreal) x (1 + i); (1 + rnominal) = (1.08) x (1.04) = 1.1232. rnominal = .1232 =
12.32%.
5. (1 + rnominal) = (1 + rreal) x (1 + i); (1.122) = (1 + rreal) x (1.036); (1 + rreal) = 1.083. rreal = .083 =
8.3%.
6. (1 + rnominal) = (1 + rreal) x (1 + i); (1.086) = (1.047)(1 + i); (1 + i) = 1.0372. i = .0372 = 3.72%

CHAPTER 6 Answers to Additional Problems and Questions


Problems:
1. 10.24%
2. 8.112%
3. 5.77%
4. 5.15%
5. 1.92%
6. 10.04%
7. 1294.80
8. 4108.90
9. 237.60
10. 4917.12
11. 969.31
12. 1696.25
13. 3752.35
14. 612.83
15. E(R) [i.e., Expected Return] on first = 11.5%;E(R) on second = 12.1%
Choose both since Expected Return > Required Return. NOTE: This problem implicitly
assumes that these two investments are of equal risk. Unless otherwise explicitly stated, for
all problems in this book, we will assume that all comparable investments are of equal risk.
16. E(R) on first = 10.2%; E(R) on second = 7.48%
Choose neither since for both E(R) < Required Return.
17. (d) is correct. For (a), PV must be less than 432 since interest rate is greater than 0.for (b), the
FV (t=1) must be greater than the PV (t=0) value since interest rate is greater than 0.for (c),
you would never pay more than the simple sum of all future cash flows (i.e., 2,300+2,300 =
4,600 < 6,600)
18. 70,661.16
19. 1,680.00; 1,915.20
20. In the following equation, solve for X (note that ^2 means raised to the second power, or int
this case, squared): 10,000 = 7,000 (1/1.13)^1 + X(1/1.13)^2 ==> X = 4,859
21. In the following equation, solve for r: 4,100 = 3,500 (1+r)^2 ==> r = 8.233%
22. Buy the two year subscription because the PV of buying a one year subscription today and
another one year subscription one year from today = 48 + 48(1/1.10) = 91.64 which is greater
than the two year subscription price of 70. This answer of course assumes that you actually
want to read this magazine for two years!
23. Assume that the two payments are due one month from today and 2 months from today.
Payoff = 791.09
24. Compare 20,000 - 2,850 = 17,150 to the PV of 20,000 two years from today.
20,000(1/1.08)^2 = 17,146.78. Since 17,150 > 17,146.78, choose (b) because it is "cheaper."

CHAPTER 7 Solutions to Assignment Problems


Assignment 7.1:
1. PV = -30000, FV = 49000, n = 5, Compute I/Y = 10.31%.
PV = -73000, FV = 128000, n = 7, Compute I/Y = 8.35%.
2. Use cash flow register: CF0 = 0, C01 = 22000, F01 = 1, C02 = 27500, F02 = 1, C03 =
33000, F03 = 1, C04 = 35000, F04 = 1; Compute NPV (with I = 6%) = 100,660.33
3. Use cash flow register to find NPV. Then find FV of this amount. Thus, CF0 = 11000, C01 =
13000, F01 = 1, C02 = 17400, F02 = 1, C03 = 12800, F03 = 1, C04 = 9600, F04 = 1; C05 =
17200, F05 = 1; Compute NPV (with I = 8%) = 66878.10. Now find the future value of this
amount 10 years from today. PV = 66878.10, I = 8, n = 10, Compute FV = 144,384.81.
Assignment 7.2:
1. PV = -28000, fv = 30000, I = 6, n = 10, Compute PMT = 1,528.26.
2. First find F of deposits. PMT = 2500, I = 8, n = 20, Compute FV20 = 114404.91. Now find
withdrawals. PV = 114404.91, n = 25, I = 8, Compute PMT = 10,717.31
3. First find NPV of all cash flows with the unknown cash flow assumed to be 0. That is, CF0 =
5000, C01 = 0, F01 = 10, C02 = -60000, F02 = 1, C03 = 0, F03 = 2, C04 = 25000, F04 = 1,
C05 = 0, F05 = 11, C06 = -1500000, F06 = 25; Compute NPV (with I = 7%) = 335884.60.
Now find PMT. PV = 335884.60, I = 7, n = 25, Compute PMT = 28,822.43.
4. First find PV20 of perpetuity = 30000/.12 = 250000 (note this is value in year 20).
CF0 = 0, C01 = 12000, F01 = 3, C02 = 17000, F02 = 4, C03 = 21000, F03 = 8, C04 = 24000,
F04 = 4; C05 = 274000, F05 = 1; Compute NPV (with I = 12%) = 154,486.56.

Assignment 7.3:
1. PV = -1, FV = 2, I = 7/2 = 3.5, Compute n = 20.1488 semi-annual periods = 10.07 years.
2. Bank A: 10.00%; Bank B: 10.04%; Bank C: 9.95%; Bank D: 9.93%; Bank E: 9.86%.
3. PMT = 140000, n = 10, I = 9, Compute PV = 898472.08. This is value need at the end of
year 20. Since first deposit will be made today, set calculator in BEGIN mode. Now, FV =
898472.08, I = 9, n = 20, Compute PMT = 16,111.89.
4. Annual payment = $8,652.62.
Year
1
2
3
4
5
6

Beginning
Balance
40,000.00
34,547.38
28,658.56
22,298.63
15,429.90
8,011.68

Interest
3,200.00
2,763.79
2,292.68
1,783.89
1,234.39
640.93

Payment

Ending
Balance

$8,652.62
$8,652.62
$8,652.62
$8,652.62
$8,652.62
$8,652.62

34,547.38
28,658.56
22,298.63
15,429.90
8,011.68
0.00

CHAPTER 7 Answers to Additional Problems and Questions


Problems:
1. a more compounding periods per year creates a larger effective interest rate.
2. e
3. 17 years
4. 4.45%
5. $10,962.37
6. d
7. a
8. 19.56%
9. $704
10. e
11. e
12. This question is misleading. This is not an annuity problem.
For example, n=10, PMT=0, r = 10, FV = 5,000, PV = ?1,927.72
n=10, PMT=0, r= 10, PV = 5,000, FV = ? 12, 968.71
Rate
0
5
10
15
20
25
30
35
40
45
50

PV
5,000.00
3,069.57
1,927.72
1,235.92
807.53
536.87
362.69
248.68
172.86
121.70
86.71

FV
5,000.00
8,144.47
12,968.71
20,227.79
30,958.68
46,566.13
68,929.25
100,532.78
144,627.33
205,423.45
288,325.20

Plot of Present Values


6,000.00
Present Value

5,000.00
4,000.00
3,000.00

Series1

2,000.00
1,000.00
0

5 10 15 20 25 30 35 40 45 50
Interest Rate

Plot of Future Values


350,000.00

Future Value

300,000.00
250,000.00
200,000.00

Series1

150,000.00
100,000.00
50,000.00
0

5 10 15 20 25 30 35 40 45 50
Interest Rate

13. $15,129.38
14. $45,349.14
15. $26.97
16. $889.23
17. 2.74%
18. 4.47%
19. $96,969.53
20. 7.10%; 7.23%
21. $31,265.66
22. 8.654%
23. $7,669.12; $18,419.93
24. $76,175.84

25. $6,714.27
Alternative 1:
I do not think solution manual gave right answer, I did in two different method and this is what I
get
CF1 = 20,000, CF1 = 0, F1= 10 (assume that t = 12 means ending of year 11)
CF2 = -25,000, F2 = 4, (ending at t = 14 or beginning at t = 15)
Cf3 = 0, f3 = 2 (assume that t = 18 means ending of year 17)
Cf4= -33,000, f4 = 4
I = 8%, NPV = - 50,257.52 (this is the spending at t =0)
You have to save for this for 10 yrs
Pv = - 70,257.52, FV = 0, N = 10, 1/Y = 8, PMT = ? 7,489.85
Alternative 2:
Pv1) PMT= 33,000; n=4; FV=0; PV=-118, 044.2006; 1/y= 8, (Solved as ANN Due) at the end of
t =17
Pv2) PMT=0; n=7; 1/y=8; FV= 118, 044.2006, PV=- 68,877.65 at the end of t = 10
PV3) PMT= 25000; n=4; i/y= 8; FV=0; Pv= -89,427.42468 (Solved as ANN Due) at the end of t
=11
PV4) Fv= 89,427.42468; n=1; 1/y= 8, PMT=0, PV= -82,803.171 at the end of t = 10
Pt 5) Pv= -20,000, n=10, FV= (68,877.65 +82,803.171), 1/y=8, PMT= -7,489.85
26. $4,520,178.42 today; $2,883,820.96 (5,082,277.89 is correct answer) in five years.
27. $263.80
28. $59,739.98
29. $9.50
30. $482.09
When PMT = 0, it does not matter if you use END or BGN mode. But I suggest you always keep
your calculator at end mode, just to avoid problems.
Cf0 = 0, cf1 = 0, f1 = 59 (beg of t = 60 implies end of t = 59)
Cf2 = -2500, f2=16, I = 1.25, NPV = - 17,322.37
PV = - 17,322.37, N = 48, 1/y = 1.25, FV = 0, PMT = ? 482.09
31. Amount of payment that goes to principal = 7757.16; Amount that goes to interest = 1,698.84
32. $925,764
33. $86,303.09
34. $17,954.13
Correct method for the solution in END mode is as follows:
PV= -3000, PMT= -3000, n=4, I/y=9, FV= 17, 954. 13
35. $276.21
36. $167,790.24
37. $5,468.21
38. $61,534.10
39. $165,918.32
40. $71,474.07
41. $871.47

As a first step, calculate the PV of the cash flows (at 10.5%): cf0 = 0, Cf1=1700, f1 = 1,
Cf2=1800, f2 = 1, cf3 = 0, f3 =1, cf4 = 2000, f4 = 1, NPV =4354. 1026
Therefore, you are 5000-4354. 1026=645.897 short of the 5000. This represents the PV
of the
missing cash flow. To calculate the actual cash flow, calculate the FV of the 645.897:
PV=645.897, N=3; I=10.5; PMT = 0; FV=??=871.47
To verify your answer, enter the 871.47 as Cf3 and calculate the PV (it works out
to 5,000)

CHAPTER 9

ASSIGNMENT 9.1
1.
a. Treat as a perpetuity.

P = Coupon/Interest Rate = 82.50/0.08 = $1,031.25

b. P = 1000/(1.0815) = $315.24
c. N = 20; PMT = 70; I/Y = 8; FV = 1000; CPT PV.

P = $901.82

2.
a. N = 60; FV = 1000; PMT = 35; I/Y = 4.5; CPT PV.

P = $793.62

b. N = 80; FV = 1000; PMT = 25; I/Y = 2.25; CPT PV.

P = $1,092.37

3. PV = -978; FV = 1000; PMT = 90; N = 14; CPT I/Y. Cost of Debt = YTM = 9.29%
4. N = 68; (i.e. 17x4 quarters) PV = -1020; FV = 1000; PMT = 20; CPT I/Y;
I/Y = 1.9467;
Thus, Cost of Debt = YTM = 1.9467 x 4 = 7.79%
ASSIGNMENT 9.2
1. Preferred should be treated as a perpetuity as it pays a perpetual stream of preferred dividends.
Price = 2.25/0.11 = $20.45
QWEs estimate of the required rate of return is higher than the actual required rate of return
investors are using to value the preferred stock.
The actual rate = 2.25/24 = 9.375%
2. Since the common stock pays constant dividends forever, the present value of the dividends is
obtained using the perpetuity formula, as in 1 above.
P = 1.50/0.09 = $16.67
3. P0 = D1 / (k-g) = 2.50/(0.10-0.03) = $35.71
4. First find P5 = D6 / (k-g) = $1(1.05)/(0.12-0.05) = 1.05/0.07 = $15.
Now P0 = 1/1.12 + 1/1.122 + 1/1.123 + 1/1.124 + (1+15)/1.125 = $12.12.
Alternative Solution: After finding P5 note that the cash flow lends itself to the use of the third
row keys on your calculator. Thus we can use our usual setup to find the present value.
PMT = 1; FV = 15; N = 5; I/Y = 12; CPT PV.
P = $12.12

ASSIGNMENT 9.3
1.
P0 = D1/(k-g) = D0(1+g)/(k-g). We can make k the subject of this equation.
k = g + D0(1+g)/P0 = 0.04 + 2(1.04)/38 = 0.04 + 0.0547 = 9.47%

2. First determine D4 = D3(1+g) = 3(1.06) = $3.18


Then P3 = D4/(k-g) = 3.18/(0.14-0.06) = $39.75
Now, P0 = 2/1.14 + 1/1.142 + (3+39.75)/1.143 = $31.38
You can also use the CF (cash flow) procedure to do the calculations in the last step:
C01 = 2; F01 =1; C02 = 1; F02 =1;

C03 = 42.75 (3+39.75); F03 = 1;

I=14.

3.
D2 = 1.7591.5)2 = 3.9375; D3 =1.75(1.52)(1.3) = 5.11875

D1 = 1.75(1.5) = 2.625;

D4 = 1.75(1.52)(1.32) = 6.654375;

D5 = 1.75(1.52)(1.32)(1.2) = 7.98525;

D6 = 1.75(1.52)(1.32)(1.2)(1.07) = 8.5442175.
P5 = D6/(k-g) = 8.5442175/(0.15-0.07) = $106.80
Then P0 = 2.625/1.15 + 3.9375/1.152 + 5.11875/1.153 + 6.654375/1.154 +
(7.98525+106.8027)/1.155 = $69.50.
Again, you can use the CF procedure (probably easier) for the last step.
4. D1 = $1(1.35) = 1.35; D2 = 1(1.352) = 1.8225; D3 = 1(1.353) = 2.460375;
D4 = 2.460375(1.2) = 2.95245; D5 = 2.95245(1.2) = 3.54294;
D6 = 3.54294(1.2) = 4.251528; D7 = D8 = D9 = = D = 4.251528.
After this, it is probably best to use the cash flow procedure for the last step:
C01 = 1.35; F01 = 1;

C02 = 1.8225; F02 = 1;

C04 = 2.95245; F04 =1;

C03 = 2.460375; F03 = 1;

C05 = 3.54294; F05 = 1; C06 = 4.251528; F06 = 999.

Use I = 15. Then the (NPV) Price = $21.71

Alternatively, the last step can be done using the analytical method:
First find P6 = D7/k = 4.251528/0.15 = $28.34.
P0 = 1.35/1.15 + 1.8225/1.152 + 2.460375/1.153 + 2.95245/1.154 + 3.54294/1.155 +
(4.251528+28.34)/1.156 = $21.71

ADDITIONAL PROBLEMS
1. PV = -1100; N = 25; PMT = 90; FV =1000; CPT I/Y.
Market interest rate = YTM = 8.06%
2. To be indifferent between calling the bond and not calling the bond, the call price of $1,075
must be equal to the present value of the remaining payments. Thus:
PV = -1075; PMT = 80; N = 25; FV = 1000; CPT 1/Y.

7.34%

3. Use the usual procedure for finding the price of a bond. Just be careful to use the appropriate
YTM for I/Y for the various times.
a. $1,000

b. $1,087.45; c. $873.08;

d. $974.69;

e. $972.97

4. FV = 1000; PV = -1092; PMT = 50; N=40; CPT I/Y. I/Y = 4.5%. Thus YTM = 9%
5. FV=1000; N=8; PMT = 0; I/Y = 4. CPT PV.

Price = $730.69

6. FV=1000; N=34; PMT = 40; I/Y = 3.625; CPT PV.

P = $1,072.62

7. FV = 1000; N = 30; PMT = 35; PV = -825; CPT I/Y. I/Y = 4.585. YTM = 9.17%
8. YTM = 9.09%
9. P = $859.16
10. First find the yield to maturity of the zero coupon bonds.
N = 30; FV =1000; PV = -99.38; PMT = 0; CPT I/Y.
YTM = 8%.
Now, the yield to maturity of the coupon bond is also 8%. Thus for the coupon bond:
N = 60; FV = 1000; PV = -886.88; I/Y = 4; CPT PMT. PMT = 35. Coupon Rate = 7%.
11. Price per share = 20000000/1000000 = $2. D = P x k = 2x0.11 = $2.20 per share.
12. P0 = D0(1+g)/(k-g) = 1.50(1.04)/(0.12-0.04) = 1.56/0.08 = $19.50
13. P0 = D0(1+g)/(k-g) = 4(1-0.05)/(0.20--0.05) = 4(0.95)/0.25 = $15.20

14. P0 = D0(1+g)/(k-g);

15 = 1(1+g)/(0.12-g); 1.80 -15g = 1+g; g = 0.8/16 = 5%

15. P0 = D0(1+g)/(k-g); 72.25 = 5.12(1.06)/(k-0.06);

k = 13.51%

16. The first dividend that begins the constant growth forever is D5. Thus, we can find P4 using:
P4 = D4 (1+g)/(k-g) = 3.5(1.08)/(0.14-0.08) = $63.
Now, either use he cash flow procedure or the analytical method to solve for P0.
P0 = 2/1.14 + 1.50/1.142 + 2.50/1.143 + (3.50+63)/1.144 = $43.97
17. First lets find the discount rate, k.

24 = 3/k. Thus k = 12.5%.

Now, with the 3 percent expected growth, the dividend one year from now, D1 = 3(1.03) = $3.09.
Therefore, P0 = 3.09/(0.125-0.03) = $32.53.
18. g = 2.14/2 1 = 7%.

P0 = 2.14/(0.26-0.07) = $11.26

Take note of the fact that the question gave some pieces of information that were irrelevant to
solving the problem.
19. First determine k, the investors required rate of return. k = 2.10/15 = 14%.
Then P0 = 2.10(1.04)/(0.14-0.04) = $21.84.
20.

D1 = 4(0.75) = 3; D2 3(0.86) = 2.58; D3 = 2.58(0.94) = 2.4252.

P3 = 2.4252(1.044)/(0.184-0.044) = 18.09. P0 = 3/1.184 + 2.58/1.1842 +


(2.4252+18.09)/1.1843 = 2.534 +1.8404 + 12.36 =16.73

CHAPTER 10

ASSIGNMENT 10.1
1.

NPV

IRR

PI

Payback
Period

Discounted
Payback Per.

$3,456.40

22.58%

1.38

2.8 years

3.22 years

$3,885.38

20.66

1.35

3.09

3.59

$5,583.82

19.48

1.24

3.11

-$2,536.31

2.62

0.80

4.84

3.44
Never Pays
Back ()

2.
PMT = $26,000,000; FV = 20,000,000; N = 10; I/Y = 12; CPT PV.
PV (Benefits) = $153,345,263.50.
Thus, NPV = PV (Benefits) PV (Costs) = 153,345,263.50 200,000,000 = - $ 46,654,736.50
IRR = 6.25%
PI = PV (Benefits)/PV (Costs) = 153,345,263.5/200,000,000 = 0.77
3. P0 = 10,000,000/(0.14-0.05) = $111,111,111.11
NPV = 111,111,111.11 100,000,000 = $11,111,111.11
The IRR is the discount rate that will make the present value of the benefits equal to the initial
cost (that is the discount rate that will make the NPV equal to zero).
100,000,000 =
10,000,000/(k-0.05);
k = IRR = 15%
PI = 111,111,111.11/100,000,000 = 1.11
ASSIGNMENT 10.2
1.
R
0
10
20
30
40
50
60
70
80

NPV
-300
86.78
250
275.15
214.29
100
-46.86
-212.80
-388.89

2.

NPV Profile for Project A

400
300
200

NPV

100
0
-100 0

10

20

30

40

50

60

70

80

90

-200
-300
-400
-500
Discount Rate
The IRRs estimated from the graph are 7% and 57%
ASSIGNMENT 10.3
1. The net cash flow per month is 12x600 1800 = $5400.
Thus, PMT = $5400; FV = $500,000; N =120; I/Y = 1; CPT PV.
PV = $527,880.21
2. The net cash flow per year = 20x6000 33000 = $87,000.
Thus, PMT = 87000; N = 12; FV = 400000; I/Y = 12; CPT PV.
PV = $641,580.59. The offer of $675,000 exceeds the present value of cash flows from
the property. Therefore SELL.
3. YTM = 8.5% = Pretax cost of debt. After tax cost of debt = (1-0.3)x(8.5%) = 5.95%.
For common stock: 20=1.5(1.05)/(k-0.05). Solve this equation for k, the
investors required rate of return.
k = 0.05 + 0.07875 = 12.875%
Cost of capital = 0.25(5.95) + 0.75(12.875) = 11.14%

4. Bond. FV = 1000; N =50; PMT = 27.5; PV = -695; CPT I/Y. I/Y = 4.2249%. Since
coupons are paid semiannually, the YTM = 2x4.2249 = 8.45%
After tax cost of debt = (1-0.3)(8.45) = 5.915%
Stock. 26.5 = 2.1(1.055)/(k-0.055). Solving for k, we have: k= 13.86%
Cost of capital = 0.6(5.915) + 0.4(13.86) = 9.09%
Additional Questions and Problems
1.
PROJECT NPV

IRR

PI

Payback Period

Disc. Payback Period

$603.58

31.79%

1.60

2.5 yrs

3.07 yrs

-$456.15

7.93%

0.92

4 yrs

Infinity

-$3974.15 6.54%

0.86

3.67 yrs

Infinity

$3807.59

1.22

3.70 yrs

4.22 yrs

16.43%

2. The project has 2 IRRs because the cash flows change signs two times. The IRRs are 48.18%
and -50.55%. To solve for the IRR, you need to solve the following equation for r.
1996/(1+r) 740/(1+r)2 -1010 = 0. You can use trial and error or you can solve using the
quadratic formula. For the quadratic, first multiply the equation through by (1+r)2 and simplify to
obtain a quadratic equation. Then use the quadratic formula.
If the cost of capital is 20%, the project should be accepted because it has a positive NPV. NPV
= $139.44.
3.

a. $100,000
b. $65,738.04
c. 18%

4. NPV = PV(Benefits) PV (costs).


(Benefits) = $21,900.

6,900 = PV(Benefits) 15000. Therefore PV

Use cash flow keys to find IRR = 24.95%.


PI = PV (benefits)/ PV(costs) = 21900/15000 = 1.46
Cost of capital (or discount rate). Since we know that the present value of the benefits equals
$21,900, we can solve for the discount rate.

PMT = 4500; N = 8; PV = -21,900; FV = 0; CPT I/Y. I/Y = cost of capital = 12.59%.


5. PI = PV (Benefits)/ PV (Costs). Thus we have 0.96 = PV (Benefits)/1000000. So,
PV(Benefits) = $960,000.
NPV = PV(Benefits) PV(Costs) = 960000 1000000 = -$40,000.
6. b.
7. e
8. a
9. NPV = $9,298.81;
positive.

IRR = 19.63%;

PI = 1.116. Accept Project as NPV is

10. B. It has the highest NPV.


11. A,B,C,D (ALL). They all have positive NPV.
12. NPV = $54.85
13. Bonds: PV = -1095; FV = 1000; PMT = 116; N =24; CPT I/Y. I/Y = Pretax cost of
debt = 10.50%. After-tax cost of debt = (1-0.3) (10.5) = 7.35%
Stock. k = 0.07+5.25(1.07)/68.25 = 15.23%
Cost of capital = 0.5(7.35) + 0.5(15.23) = 11.29%
14. Bonds: PV = -8785; N = 60; FV = 10000; PMT = 390; CPT I/Y. I/Y = 4.487%. Thus
the YTM = Pre-tax cost of debt = 2(4.487) = 8.97%.
After-tax cost of debt = (1-0.30)(8.97%) = 6.28%.
Stocks: 33.75 = 2.90(1.0375)/(k-0.0375). Solving for k we get k = 12.66%, the cost of
equity.
Thus, cost of capital = 0.8(6.28) + 0.2 (12.66) = 7.56%.

CHAPTER 11
Assignment 11.1:
1.
Determine Net Cash Flow: NCF = (2,000,000 Depreciation)(1 T) + Depreciation
NCF = (2,000,000 1,000,000)(.7) + 1,000,000 = 1,700,000.
CF0 = -10000000; C01= 1700000; F01=10; Compute NPV (with I = 13) = -775,386.09.
2.

Project A: CF0 = -80000; C01=18000; F01=8; Compute NPV (with I = 10) = 16,028.67. With same
figures in CF register, compute IRR = 15.29%
Project B: CF0 = -40000; C01=10000; F01=8; Compute NPV (with I = 10) = 13,349.26. With same
figures in CF register, compute IRR = 18.62%
Note that NPV indicates that we should accept Project A and IRR indicates that we should accept
Project B. ALWAYS CHOOSE THE PROJECT WITH THE HIGHEST NPV, so choose Project A.

3.

Enter all values in cash flow register and compute:


NPV (A) = 4497.54; IRR(A) = 19.31%
NPV (B) = 7937.38; IRR(B) = 28.97%
Choose Project B because higher NPV.

4.

If you construct graph correctly, you should find that the two lines cross at a discount rate of
approximately 16.25%. Note that with I = 16.25, NPV (P) = 1249.76 and NPV (Q) = 1249.77.
At rates above 16.25%, NPV(P) > NPV(Q)
At rates above 16.25%, NPV(P) < NPV(Q).

Assignment 11.2:
1.
(Assume tax rate = 0). First compute the NPV of cost of each machine.
X100: CF0=-45000; C01=-5000; F01=4; C02=-5000+6000=1000; F02=1; Compute NPV (I=12) =
-59619.32. Note that this is NPV of costs.
X1300: CF0=-65000; C01=-2000; F01=6; C02=-2000+10000=8000; F02=1; Compute NPV (I=12) =
-69604.02. Note that this is NPV of costs.
Because these two projects have different lives, to compare we must now compute EAC.
EAC(X100): PV = -59619.32; N=5 (i.e., project life); I/Y=12, CPT PMT = 16,538.98.
EAC(X1300): PV = -69604.02; N=7 (i.e., project life); I/Y=12, CPT PMT = 15,251.48.
Thus EAC (X100) = 16,538.98. EAC (X1300) = 15,251.48. Choose X1300 because it has a lower
equivalent annual cost.

2.

Initial Cash Outflow:


Buy Machine
Decrease in NWC
CF0

-31M
+ 2M
-29M

Note that annual depreciation = (31-3)/20 = 1.4M


Annual Operating NCF = (Rev Costs Dep)(1-T) + Dep
= [(5)(.9) (.6)(5)(.9) - .6 1.4](.7) + 1.4 = 1.26M per year for 20 years
Final Year Non-Operating Cash Flow:
Sell Machine for Scrap Value
Replace NWC
Total

3M
-2M
1M

To Compute NPV: CF0 = -29, C01 = 1.26; F01 = 19; C02 = 2.26; F02 = 1;
CPT NPV (with I = 11) = -18.842M
Assignment 11.3:
Q36: CF0 = -3M; C01 = .6M; F01 = 8; CPT NPV = 200,955.72.
Z96: CF0 = -4M; C01 = .7M; F01 = 10; CPT NPV = 301,196.97.
Since these projects have different lives, to compare, must compute Equivalent Annual Annuity.
EAA(Q36):
EAA(Z96):

PV = 200955.72; I = 10; N = 8; CPT PMT = EAA = 37,667.95.


PV = 301196.97; I = 10; N = 10; CPT PMT = EAA = 49018.42.

Choose Z96, since it has higher EAA.


Problems:
1.
Must compute EAA of each project. Rank of EAA: D, C, A, B.
2.
Relevant Opportunity cost
Not relevant Sunk cost
Relevant Initial cost
Not relevant Sunk cost
Relevant Incremental expense
Not relevant Sunk cost
The incremental revenue of 93% of $2 million is relevant.
The tax savings associated with depreciation is relevant. Specifically, 36% of incremental
depreciation.
Relevant Incremental expense
Not relevant No incremental cost
Relevant Incremental
Not relevant must be paid regardless of whether project is accepted or rejected. Therefore, not
incremental.
3.
The easiest way to do this problem is to assume the company can sell radiators for $150 per
radiator. Then:

NPV (Buy from Supplier) = 13,477,081. NPV (Manufacture) = 9,439,730.

???

Project Buy: CF0=0, CF1=(150-120)*100,00*(1-0.3)=2.1 million, F1=10, NPV=13,477,081


Project Manufacture:
(150-80)*100,000=7 mil , 7 mil 0.5m (depreciation)=6.5 million, 6.5*(1-0.3)=4.55 mil
4.55 mil + 0.5 mil (depreciation)=5.05 mil
CF0=-5 million, CF1=5.05 mil, F1=10, NPV= 27,409,171
So make the radiators.
4.
5.
6.
7.
8.
9.
10.
11.
12.
13.

EAC(A) = 1160.76; EAC(B) = 1813.90 Choose project with lower equivalent annual cost.
NPV(Superior) = 17,802.68; NPV (Peerless) = 16,816.12 Choose project with highest NPV.
IRR(A) = 34.90%; NPV(A) = 5,849.33
IRR(B) = 31.61%; NPV(B) = 10,490.40
Crossover rate = 7.167%; NPV(X) = NPV(Y) = 6720.60.
a. Yes incremental; b. No Sunk; c. No financing cost (captured in NPV analysis);
d. Yes incremental; e. Yes incremental.
Note that annual NCF = Net income + Depreciation = 700,000 per year. NPV = 227,462.17.
3,690.93
Note that annual NCF = Net income + Depreciation = 6.55M per year. NPV = -5.879M.
CF0 = (12,000,000); CF1 through CF10 = 2,880,000; No final year non-operating cash flow. NPV =
1,919,695.
CF0 = (26,800,000); Operating net cash flow = 26,800,000; Final year non-operating cash flow =
2,200,000. NPV = 97,762,829.
(56,004.33)
For this problem, assume that the cost of capital is 14%. Then, NPV = (320,096). Reject.

14.
15.
16.
Step 1: Initial Investment in the Project

Opportunity cost of the land


-8M
Buy knives, blades, etc.
-5.7M
New Building construction
-70M
Increase in Net Working capital -10M
Total Year 0 Cash Flow
-93.7M
Step 2: Annual Operating Cash Flow
Note that Incremental revenue = (3000)(300)(30) = 27M
and Depreciation = (5.7-.7)/25 + 70/25 = 3.0M
NCF = (27M - 3.2M - 4.5M - 2.3M - 3.0M)(.6) + 3.0M = 11.4M for 25 years.
Step 3: Final Year non-operating Cash Flow
Sell land
8M
Sell equipment
.7M
Recover NWC
10M
Total
18.7M

To compute NPV:
CF0=-93.7; C01 = 11.4; F01=24; C02=11.4+18.7=30.1; F02 = 1; CPT NPV = -14.642M.

You might also like